2
$\begingroup$

The bound formula is:

$$H(M,N,n,k) \leq (( \frac{p}{p+t} )^{p+t} (\frac{1-p}{1-p-t})^{1-p-t})^n$$

So how to proof it?

Some papers discuss it: this one, that one.

They say the proof is in the following paper: The tail of the hypergeometric distribution. Discrete Mathematics

But I can not find the online version.

  • 0
    The paper is online here, but you need a university subscription. http://dx.doi.org/10.1016/0012-365X(79)90084-02011-02-14
  • 0
    Please don't post the same questions (more or less) simultaneously here and on MathOverflow. I suggest the following procedure: Ask here first and if you didn't get any helpful answer in a few *days* you might consider asking on MathOverflow. http://mathoverflow.net/questions/55402/2011-02-14
  • 0
    Also you have the MR entry http://www.ams.org/mathscinet-getitem?mr=MR05349462011-02-14
  • 0
    I'm sending you the paper per email, check your spam folder in case it isn't there in a few minutes. @Willie: I'm notifying you in order to avoid double action.2011-02-14
  • 2
    That, plus the fact that the inequality is a straightforward consequence of the usual exponential inequalities applied to sums of random variables. That is, coming back to the *physical* interpretation of the hypergeometric distribution, one considers $S_n=X_1+\cdots+X_n$ where $X_i$ is Bernoulli and $X_i=1$ iff the ball drawn at time $i$ is white. The only twist here is that the $X_i$s are not independent. Fortunately, they are *negatively correlated*, hence the upper bound one would get by assuming independence is still valid.2011-02-14
  • 0
    @Didier: Ah, that clarifies a lot. Thanks!2011-02-14
  • 0
    @Didier: Could you explain why when the random variable is negatively correlated, the independence assumption is still valid.2011-02-14
  • 1
    @Fan: That would be absurd and is not what I wrote. Rather, for every negatively correlated $U$ and $V$, $E(UV)\le E(U)E(V)$. (And if you merely look at the definition, you will see that it is meaningless to say that a random variable is negatively correlated, on its own, all alone...)2011-02-14
  • 0
    @Didier Thank you for your explaination.2011-02-14

1 Answers 1

2

This question has been answered to the satisfaction of the OP in the comments.